On computing a conditional expectation $E[Y|X]$ where $Y$ is not known.












0












$begingroup$


Preface:

This question is based on the answer to this question given in the comments.



The problem:




Consider the Lebesgue probability space on the interval $[0,1)$. (I.e. the state space is $Ω = [0, 1)$, the $sigma$-field is the set of Lebesgue measurable sets and the measure is the Lebesgue measure.) We define the random variable $X$ as:
$$X(w)=begin{cases} 2w &, 0leq w < 1/2 \ 2w−1 &, 1/2leq w<1 end{cases}$$
Compute the conditional expectation $E(Y |X)$ where $Y : [0, 1) to mathbb{R}$ is a measurable function.




My question:



Why can we write $$E(Ymid X)(omega)=frac{Y(omega)+Y(omega+frac12)mathbf 1_{2omega<1}+Y(omega-frac12)mathbf 1_{2omegageqslant1}}2$$ ? What theorem are we appealing to?



EDIT: I think an answer was given to a similar question here.










share|cite|improve this question











$endgroup$












  • $begingroup$
    The title does not agree with the equation you have written Are you computing $E(X|Y)$ or $E(Y|X)$?
    $endgroup$
    – Kavi Rama Murthy
    Oct 7 '18 at 23:41






  • 1




    $begingroup$
    @KaviRamaMurthy from the linked question it is $E(Y mid X)$
    $endgroup$
    – Henry
    Oct 7 '18 at 23:43










  • $begingroup$
    @KaviRamaMurthy I will edit, sorry for the typo.
    $endgroup$
    – Monolite
    Oct 8 '18 at 10:39






  • 1




    $begingroup$
    @Did Maybe you could take a look at this question since it comes from a comment of yours. In any case thanks for all the answers and your activity on mathstack!
    $endgroup$
    – Monolite
    Oct 23 '18 at 22:43
















0












$begingroup$


Preface:

This question is based on the answer to this question given in the comments.



The problem:




Consider the Lebesgue probability space on the interval $[0,1)$. (I.e. the state space is $Ω = [0, 1)$, the $sigma$-field is the set of Lebesgue measurable sets and the measure is the Lebesgue measure.) We define the random variable $X$ as:
$$X(w)=begin{cases} 2w &, 0leq w < 1/2 \ 2w−1 &, 1/2leq w<1 end{cases}$$
Compute the conditional expectation $E(Y |X)$ where $Y : [0, 1) to mathbb{R}$ is a measurable function.




My question:



Why can we write $$E(Ymid X)(omega)=frac{Y(omega)+Y(omega+frac12)mathbf 1_{2omega<1}+Y(omega-frac12)mathbf 1_{2omegageqslant1}}2$$ ? What theorem are we appealing to?



EDIT: I think an answer was given to a similar question here.










share|cite|improve this question











$endgroup$












  • $begingroup$
    The title does not agree with the equation you have written Are you computing $E(X|Y)$ or $E(Y|X)$?
    $endgroup$
    – Kavi Rama Murthy
    Oct 7 '18 at 23:41






  • 1




    $begingroup$
    @KaviRamaMurthy from the linked question it is $E(Y mid X)$
    $endgroup$
    – Henry
    Oct 7 '18 at 23:43










  • $begingroup$
    @KaviRamaMurthy I will edit, sorry for the typo.
    $endgroup$
    – Monolite
    Oct 8 '18 at 10:39






  • 1




    $begingroup$
    @Did Maybe you could take a look at this question since it comes from a comment of yours. In any case thanks for all the answers and your activity on mathstack!
    $endgroup$
    – Monolite
    Oct 23 '18 at 22:43














0












0








0


1



$begingroup$


Preface:

This question is based on the answer to this question given in the comments.



The problem:




Consider the Lebesgue probability space on the interval $[0,1)$. (I.e. the state space is $Ω = [0, 1)$, the $sigma$-field is the set of Lebesgue measurable sets and the measure is the Lebesgue measure.) We define the random variable $X$ as:
$$X(w)=begin{cases} 2w &, 0leq w < 1/2 \ 2w−1 &, 1/2leq w<1 end{cases}$$
Compute the conditional expectation $E(Y |X)$ where $Y : [0, 1) to mathbb{R}$ is a measurable function.




My question:



Why can we write $$E(Ymid X)(omega)=frac{Y(omega)+Y(omega+frac12)mathbf 1_{2omega<1}+Y(omega-frac12)mathbf 1_{2omegageqslant1}}2$$ ? What theorem are we appealing to?



EDIT: I think an answer was given to a similar question here.










share|cite|improve this question











$endgroup$




Preface:

This question is based on the answer to this question given in the comments.



The problem:




Consider the Lebesgue probability space on the interval $[0,1)$. (I.e. the state space is $Ω = [0, 1)$, the $sigma$-field is the set of Lebesgue measurable sets and the measure is the Lebesgue measure.) We define the random variable $X$ as:
$$X(w)=begin{cases} 2w &, 0leq w < 1/2 \ 2w−1 &, 1/2leq w<1 end{cases}$$
Compute the conditional expectation $E(Y |X)$ where $Y : [0, 1) to mathbb{R}$ is a measurable function.




My question:



Why can we write $$E(Ymid X)(omega)=frac{Y(omega)+Y(omega+frac12)mathbf 1_{2omega<1}+Y(omega-frac12)mathbf 1_{2omegageqslant1}}2$$ ? What theorem are we appealing to?



EDIT: I think an answer was given to a similar question here.







probability probability-theory expected-value






share|cite|improve this question















share|cite|improve this question













share|cite|improve this question




share|cite|improve this question








edited Dec 2 '18 at 16:03







Monolite

















asked Oct 7 '18 at 21:33









MonoliteMonolite

1,5412925




1,5412925












  • $begingroup$
    The title does not agree with the equation you have written Are you computing $E(X|Y)$ or $E(Y|X)$?
    $endgroup$
    – Kavi Rama Murthy
    Oct 7 '18 at 23:41






  • 1




    $begingroup$
    @KaviRamaMurthy from the linked question it is $E(Y mid X)$
    $endgroup$
    – Henry
    Oct 7 '18 at 23:43










  • $begingroup$
    @KaviRamaMurthy I will edit, sorry for the typo.
    $endgroup$
    – Monolite
    Oct 8 '18 at 10:39






  • 1




    $begingroup$
    @Did Maybe you could take a look at this question since it comes from a comment of yours. In any case thanks for all the answers and your activity on mathstack!
    $endgroup$
    – Monolite
    Oct 23 '18 at 22:43


















  • $begingroup$
    The title does not agree with the equation you have written Are you computing $E(X|Y)$ or $E(Y|X)$?
    $endgroup$
    – Kavi Rama Murthy
    Oct 7 '18 at 23:41






  • 1




    $begingroup$
    @KaviRamaMurthy from the linked question it is $E(Y mid X)$
    $endgroup$
    – Henry
    Oct 7 '18 at 23:43










  • $begingroup$
    @KaviRamaMurthy I will edit, sorry for the typo.
    $endgroup$
    – Monolite
    Oct 8 '18 at 10:39






  • 1




    $begingroup$
    @Did Maybe you could take a look at this question since it comes from a comment of yours. In any case thanks for all the answers and your activity on mathstack!
    $endgroup$
    – Monolite
    Oct 23 '18 at 22:43
















$begingroup$
The title does not agree with the equation you have written Are you computing $E(X|Y)$ or $E(Y|X)$?
$endgroup$
– Kavi Rama Murthy
Oct 7 '18 at 23:41




$begingroup$
The title does not agree with the equation you have written Are you computing $E(X|Y)$ or $E(Y|X)$?
$endgroup$
– Kavi Rama Murthy
Oct 7 '18 at 23:41




1




1




$begingroup$
@KaviRamaMurthy from the linked question it is $E(Y mid X)$
$endgroup$
– Henry
Oct 7 '18 at 23:43




$begingroup$
@KaviRamaMurthy from the linked question it is $E(Y mid X)$
$endgroup$
– Henry
Oct 7 '18 at 23:43












$begingroup$
@KaviRamaMurthy I will edit, sorry for the typo.
$endgroup$
– Monolite
Oct 8 '18 at 10:39




$begingroup$
@KaviRamaMurthy I will edit, sorry for the typo.
$endgroup$
– Monolite
Oct 8 '18 at 10:39




1




1




$begingroup$
@Did Maybe you could take a look at this question since it comes from a comment of yours. In any case thanks for all the answers and your activity on mathstack!
$endgroup$
– Monolite
Oct 23 '18 at 22:43




$begingroup$
@Did Maybe you could take a look at this question since it comes from a comment of yours. In any case thanks for all the answers and your activity on mathstack!
$endgroup$
– Monolite
Oct 23 '18 at 22:43










0






active

oldest

votes











Your Answer





StackExchange.ifUsing("editor", function () {
return StackExchange.using("mathjaxEditing", function () {
StackExchange.MarkdownEditor.creationCallbacks.add(function (editor, postfix) {
StackExchange.mathjaxEditing.prepareWmdForMathJax(editor, postfix, [["$", "$"], ["\\(","\\)"]]);
});
});
}, "mathjax-editing");

StackExchange.ready(function() {
var channelOptions = {
tags: "".split(" "),
id: "69"
};
initTagRenderer("".split(" "), "".split(" "), channelOptions);

StackExchange.using("externalEditor", function() {
// Have to fire editor after snippets, if snippets enabled
if (StackExchange.settings.snippets.snippetsEnabled) {
StackExchange.using("snippets", function() {
createEditor();
});
}
else {
createEditor();
}
});

function createEditor() {
StackExchange.prepareEditor({
heartbeatType: 'answer',
autoActivateHeartbeat: false,
convertImagesToLinks: true,
noModals: true,
showLowRepImageUploadWarning: true,
reputationToPostImages: 10,
bindNavPrevention: true,
postfix: "",
imageUploader: {
brandingHtml: "Powered by u003ca class="icon-imgur-white" href="https://imgur.com/"u003eu003c/au003e",
contentPolicyHtml: "User contributions licensed under u003ca href="https://creativecommons.org/licenses/by-sa/3.0/"u003ecc by-sa 3.0 with attribution requiredu003c/au003e u003ca href="https://stackoverflow.com/legal/content-policy"u003e(content policy)u003c/au003e",
allowUrls: true
},
noCode: true, onDemand: true,
discardSelector: ".discard-answer"
,immediatelyShowMarkdownHelp:true
});


}
});














draft saved

draft discarded


















StackExchange.ready(
function () {
StackExchange.openid.initPostLogin('.new-post-login', 'https%3a%2f%2fmath.stackexchange.com%2fquestions%2f2946350%2fon-computing-a-conditional-expectation-eyx-where-y-is-not-known%23new-answer', 'question_page');
}
);

Post as a guest















Required, but never shown

























0






active

oldest

votes








0






active

oldest

votes









active

oldest

votes






active

oldest

votes
















draft saved

draft discarded




















































Thanks for contributing an answer to Mathematics Stack Exchange!


  • Please be sure to answer the question. Provide details and share your research!

But avoid



  • Asking for help, clarification, or responding to other answers.

  • Making statements based on opinion; back them up with references or personal experience.


Use MathJax to format equations. MathJax reference.


To learn more, see our tips on writing great answers.




draft saved


draft discarded














StackExchange.ready(
function () {
StackExchange.openid.initPostLogin('.new-post-login', 'https%3a%2f%2fmath.stackexchange.com%2fquestions%2f2946350%2fon-computing-a-conditional-expectation-eyx-where-y-is-not-known%23new-answer', 'question_page');
}
);

Post as a guest















Required, but never shown





















































Required, but never shown














Required, but never shown












Required, but never shown







Required, but never shown

































Required, but never shown














Required, but never shown












Required, but never shown







Required, but never shown







Popular posts from this blog

Plaza Victoria

In PowerPoint, is there a keyboard shortcut for bulleted / numbered list?

How to put 3 figures in Latex with 2 figures side by side and 1 below these side by side images but in...